acute gastroenteritis

69
Medex Objectives Winter 2003 MEDEX Northwest Physician Assistant Objectives Home: http://faculty.washington.edu/alexbert/MEDEX/ Last updated 7 Dec 2003 MCHPediatric Gastroenteritis 1. Gastroenteritis (Acute Diarrhea)- Discuss incidence, pathophysiology, clinical presentation, work-up, laboratory tests, and treatment. Anonymous incidence: in US, acute diarrhea strikes each child under 3 y.o. 1.3-2.3X/yr. An avg. of 220,000 kids under 5 are hospitalized yearly, with 900,000+ hospital days. 9% of all hospitalizations of kids under 5 secondary to diarrhea. 300 kids under 5 die each yr from diarrhea and dehydration. pathophysiology : 5 mechanisms exist: A. Osmotic diarrhea occurs with osmotically active particles in intestine. B. Secretory d. results from inhibition of ion absorption or stimulation of ion secretion. C. Deletion or inhibition of a normal active ion absorptive process (congenital or acquired). D. Inflammation usu. results from decrease in functional areas of bowel. E. Abnormal intestinal motility. clinical presentation : Usually last few days to a week. Diarrhea lasting more than 2 weeks is something else (malabsorption, malnutrition or both). Mostly caused by intestinal infections or food intolerance; other poss. causes include food poisoning, inflammatory disorders, iatrogenic agents (antibiotics, laxatives). work-up :

Upload: abel-quisquis

Post on 02-Dec-2014

301 views

Category:

Documents


3 download

TRANSCRIPT

Page 1: Acute Gastroenteritis

Medex Objectives Winter 2003

MEDEX Northwest Physician Assistant Objectives Home: http://faculty.washington.edu/alexbert/MEDEX/

Last updated 7 Dec 2003

MCHPediatric Gastroenteritis

 

1.         Gastroenteritis (Acute Diarrhea)- Discuss incidence, pathophysiology, clinical presentation, work-up, laboratory tests, and treatment.

Anonymous

incidence:   in US, acute diarrhea strikes each child under 3 y.o. 1.3-2.3X/yr.  An avg. of 220,000 kids under 5 are hospitalized yearly, with 900,000+ hospital days.  9% of all hospitalizations of kids under 5 secondary to diarrhea.  300 kids under 5 die each yr from diarrhea and dehydration.

pathophysiology: 5 mechanisms exist:  A. Osmotic diarrhea occurs with osmotically active particles in intestine.  B. Secretory d. results from inhibition of ion absorption or stimulation of ion secretion.  C. Deletion or inhibition of a normal active ion absorptive process (congenital or acquired).  D. Inflammation usu. results from decrease in functional areas of bowel.  E.  Abnormal intestinal motility.

clinical presentation:  Usually last few days to a week.  Diarrhea lasting more than 2 weeks is something else (malabsorption, malnutrition or both).  Mostly caused by intestinal infections or food intolerance; other poss. causes include food poisoning, inflammatory disorders, iatrogenic agents (antibiotics, laxatives).

work-up:  

Hx: including dietary record, & changes in diet must be correlated with stool frequency and form.  Family health, travel to place with contaminated water system, endemic infections, day care, foods recently ingested are all important factors.  Thorough fam. hx, GI ROS important to elicit.  Short incubation, short duration (<24 hrs.) illnesses are usu. due to ingestion of a preformed toxin.  Duration of several days indicates infection w/ agent producing enterotoxin or invasion more likely.  Dampness of recent diapers (6-8 hrs.) useful to assess hydration.  

PE:  assess airway/ventilation.  Then focus on hydration status.  Vitals, esp. BP & HR, are monitored.  Skin turgor, moistness of mucosal membranes and tearing are useful signs.  Check also for extraintestinal infection or systemic disease, determine if there is systemic toxicity.  

laboratory tests:   For uncomplicated diarrhea without evidence of dehydration or toxicity, no extensive eval. is needed.  More aggressive for toxic, dehydrated pt.  Examine stool: most

Page 2: Acute Gastroenteritis

important.  Check color, consistency, odor, presence/absence of blood or mucus.  Check pH.  Culture if results will alter Tx.  Culture also in kids <3 mos., <1 yr if toxic, also those with hemoglobinopathies or immunocompromised states.  Proctosigmoidoscopy is indicated for pts. w/ negative cultures and persistent symptoms, esp. bloody diarrhea.  

treatment:  focuses on oral rehydration therapy (ORT), w/ an objective of restoring or maintaining adequate hydration and electrolyte balance, and ensuring pt.’s nutritional status.  

Anonymous Dershewitz 582-589

Incidence- diarrheal disease continues to be one of the primary causes of morbidity and mortality in the world, where its incidence is estimated at 2.6 episodes per child per year in children younger than 5 years, and is associated with 3 to 5 million deaths per year.  Its incidence in the United States with children under the age of 3 is 1.3 to 2.3 episodes per child per year.  An average of 220,000 children younger than 5 years are hospitalized each year with gastroenteritis.  Up to 20% of acute care visits to metropolitan hospitals are related to diarrheal illness.  Approximately 300 children under the age of 5 die each year of diarrhea and dehydration.

 

Pathophsiology- there is five mechanisms of diarrheal production: o Osmotic Diarrhea- occurs when osmoticlly active particles are present in the

intestinal lumen.  Examples include the dumping syndrome, lactase deficiency, and overfeeding.

o Secretory Diarrhea- results from inhibition of ion absorption or stimulation of ion secretion.  Examples include diarrheas secondary to bacterial exotoxins and diarrheas secondary to substances produced by the body that activate secretion, such as gastrin in Zollinger-Ellison syndrome.

o Deletion or inhibition of a normal active ion absorption process- can be congenital as in congenital chloridorrhea, or acquired, such as in bile salt deficiency and pancreatic enzyme deficiencies.

o Inflammation- usually is secondary to the decrease in the anatomical or functional areas, such as occurs in mucosal disease like celiac sprue, after bacterial invasion, or after bowel resection.

o Abnormal intestinal motility- abnormally reduced peristalsis may allow bacterial overgrowth; rapid motility may reduce contact time between the small bowel mucosa and its contents.

 

Clinical Presentation- acute diarrhea is usually self-limited illness lasting a few days to a week, whereas persistent diarrhea typically persists longer than 2 weeks and may be

Page 3: Acute Gastroenteritis

associated with mal-absorption, malnutrition, or both.  Almost all acute diarrhea is caused by intestinal infections or food intolerance.  Bacterial organisms that invade the mucosa often cause fever, and if the colon is primarily involved, abdominal pain, tenesmus, fecal urgency, and stools with blood and mucus are common.  Patients with secretory diarrhea have abdominal cramps with the passage of low to moderate number of large volume stools.

 

Work-up- a thorough dietary record with changes in the diet should be correlated with stool frequency and form.  Information about other family members with gastrointestinal complaints, endemic infections, recent travel, time spent in day care, or food recently ingested are also important to elicit during the history.  A family history of chronic diarrhea, cystic fibrosis, celiac sprue, inflammatory bowel disease, and other chronic conditions should be noted.  Other symptoms like fever, vomiting or abdominal cramps should be described.  The number of wet diapers in a 6-8 hour period may also be useful.

 

Laboratory Tests- examination of the stool is the single most important step in defining the diarrheal illness.  Stoll should be observed fore color, consistency, odor, and the presence or absence of blood or mucus.  The stool PH should be attained and a clinitest should be preformed.  A culture of the stool should be reserved for those patients in whom the results will alter the therapeutic plan.  Proctosigmoidoscope with biopsy is limited, but should be preformed when pseudomembranous colitis is suspected.

 

Treatment- supportive therapy for acute diarrhea is oral rehydration therapy (ORT).  The object of ORT should be the restoration or maintenance of adequate hydration and electrolyte balance.  The traditional procedure for rehydration of ill children with acute diarrhea is hospitalizing them and administering intravenous fluid therapy while fasting the patients for variable periods.  After rehydration, variable types of clear fluids or oral rehydration solutions are administered and intravenous fluids are slowly weaned.  Antibiotics may be given but should be reserved for septic children.

L.H. Emer.Med. pgs.838-843

I am going to post a lot about this, however, you should really look at these pages because there is a LOT of info on this.

 

Infectious diarrhea may be either inflammatory (dysentery) or noninflammatory.

Page 4: Acute Gastroenteritis

Inflammatory diarrhea is commonly due to bacterial or parasitic infections/infestations.  These organisms tend to localize in the colon or distal small bowel and result in dysentery, which is characterized by frequent bowel movements containing blood, mucous, or pus.  Noninflammatory diarrhea is caused by viruses and parasites that localize to the small bowel, or by toxin producing bacteria.  In these cases, the diarrhea is profuse, watery, and commonly assoc. with nausea and vomiting.  Fever is less likely.  The term gastroenteritis is applied to this group of infections, although the stomach is rarely involved.

 

Epidemiology:

In the U.S. children younger than age 3 have 1.3-2.3 episodes of diarrhea/yr.  More in kids attending daycare.  Up to 1/5 of all acute care outpatient visits to hospitals are due to infants or children with gastroenteritis.  9% of all hospitalizations of children younger than 5 are for diarrhea.  In 10 % of cases, clinical dehydration may occur, and is life threatening in 1%.

Pathogenic viruses, bacteria, or parasites may be isolated from nearly 50% of children with diarrhea.  Viral infection is the most common.  Bacterial infections may be isolated in 1-4% of cases.

 

Pathophysiology:

Viral pathogens cause acute GE by tissue invasion and a directly cytopathic effect to small intestinal villous cells which causes villous damage and decreased intestinal absorption of nutrients, electrolytes, and water, resulting in watery diarrhea.  Villous injury also results in reduced disaccharidase levels and diminished total mucosal glucose-coupled sodium transport.  End result is a decrease in intestinal water absorption.  The volume of fluid delivered from the lumen of the damaged small intestine exceeds the colons limited ability for fluid absorption, and the net result is watery diarrhea.

 

Bacteria cause diarrhea by a variety of mechanisms, including production of enterotoxins and cytotoxins and invasion of the mucosal absorptive surface.  Bacterial toxins may also be ingested directly in food.  Most common are heat stable toxins produced by staph aureus products.  Bacillus cereus produces a heat soluble toxin typically ingested with boiled or fried rice.  

 

Parasitic infestations may cause diarrhea by a variety of mechanisms similar to those discussed for viral GE.

 

Page 5: Acute Gastroenteritis

Clinical presentation:

Depends on type of infectious agent causing the problem.  Most common presenting symptoms include watery diarrhea, fever, vomiting, abdominal pain, and dehydration.

 

Evaluation, workup:

Obtain a medical history carefully, and selectively choose lab tests according to your suspicion of the causative organism.

 

Viral infections—no diagnostic tests indicated.  Evaluate for dehydration and treat accordingly.

 

Suspected bacterial etiology with presence of fever, abrupt onset, more than 4 stools/day, guiac positive:  consider fecal WBC, stool culture, or serologic testing.  Assess for dehydration and treat according to level of dehydration.

 

Treatment:

Viral cause—treat the dehydration.  

Bacterial—obtain appropriate cultures and treat dehydration while waiting for culture results.  Consider empirical antibiotic therapy while awaiting culture results if :

              1.  fecal leukocyte positive

              2.  bloody diarrhea, fever, abdominal pain

              3.  dehydration or more than 8 stools/24 hrs

              4.  immunocompromised

              5.  hospitalization required

 

The majority of children with diarrhea and vomiting can be treated  with ORS (oral rehydration solution).  It successfully rehydrates 90% of children in whom it is used.  Pedialyte and popsicles

Page 6: Acute Gastroenteritis

are used for this.    IV rehydration with Ringers Lactate or normal saline may also be used.  (see question 2 for specifics)

 

Reinstatement of food should begin after the 4 hour rehydration phase is completed and never delayed more than 24 hours.  Breast feeding should be routinely continued for infants with acute GE.  BRAT diet (bananas, rice, applesauce, toast) can be recommended, however, it does not provide adequate energy, fat, or protein.  Encourage lean meats, yogurt, fruits, vegetables, and complex carbohydrates.

 

Indications for hospital admission:

All infants who appear toxic should be admitted.  Patients with 10% dehydration, intractable vomiting, and altered consciousness should be given an infusion of normal saline or Ringers Lactate, regardless of serum osmolality, and admitted.  Infants who are malnourished should be admitted.  Children in high-risk social situations should also be admitted for treatment.  These include the single, often teenage parent without an intact support system, or parents who are homeless or unable to provide appropriate fluids to the child.

D. Higbee, Tintinalli pg. 839

In the U.S. children younger than 3 years of age have 1.3 to 2.3 episodes of diarrhea each year.  Pathophysiology: Viral pathogens cause acute gastroenteritis by tissue invasion and a directly cytopathic effect to small intestinal villous cells.  As a consequence, there is villous damage and decreased intestinal absorption of nutrients, electrolytes, and water, resulting in a watery diarrhea.  Bacteria cause diarrhea by a variety of mechanisms, including production of enterotoxins and cytotoxins and invasion of the mucosal absorptive surface.  Clinical Presentation: Evaluation of a childs state of hydration is most important.  Child may have an ill appearance, capillary refill longer than 3 seconds, dry mucous membranes, and absent tears.  Bacteria that invade the mucosa of the terminal ileum and colon can cause dysentery, which is characterized by frequent bowel movements that contain blood, mucus, or pus.  The diarrhea is often accompanied by fever, tenesmus, and painful defecation.  Work-up: A careful medical Hx, PE, and selective lab testing, assess dehydration.  Labs:  Cultures and immunoassays of stool for the presence of the enteric pathogens.  Treatment: Oral Rehydration Solution (ORS), IVfluids, if bacterial, antibiotics.  Avoid fatty or high carb foods.

Anonymous p. 630, Current Pediatrics; p. 838-848, Tintinalli

            Incidence:

            Mainly in infants between 3 & 15 months and is seen in the largest amounts during the winter months.  In children <3 years of age average 1.3 to 2.3 episodes per year.

Page 7: Acute Gastroenteritis

 

            Pathophysiology:

            Viral pathogens invade the tissues and cause a cytopathic effect on small intestinal villous cells.  This results electrolytes, and water.  Villous injury also results in ↓ disaccharide levels and diminished total mucosal glucose-coupled sodium transport.  This leads to ↓ intestinal H2O absorption.  

           

            Bacterial pathogens can cause GE by a variety of mechanisms including enterotoxins and cytotoxins and invasion of the mucosal absorption surface.

 

            Clinical Presentation:

            Ill general appearance, cap refill > than 3 sec., dry mucous membranes, and absent tears are good indicators.  The presence of 2 or more signs predicts > 5% dehydration and 3 or more predicts a > 10% dehydration.

 

            Bacteria may cause dysentery characterized by frequent bowl movements that contain blood, mucus, or pus.  The diarrhea is often accompanied by fever, tenesmus (spasm of the anus or bladder) and painful defecation.

 

            Work-up:

            Careful medical history and selective lab testing.

 

            Lab tests:

            Routine stool culture for bacterial pathogens.  Enzyme immunoassays to test stool for rotavirus, enteric adenovirus, and astrovirus.  Fecal WBC serologic testing as well.

 

            Treatment:

Page 8: Acute Gastroenteritis

Use oral rehydration fluid (Pedialyte).  For IV rehydration between 20 ml/kg and 40 ml/kg of ringers lactate solution or normal saline may be given over 1-3 hours.  Antibiotic therapy does not affect the clinical course in most cases.  However, if dealing with persistent symptoms and other signs of infection they may be treated with antibiotics as indicated by stool sample.  Also infants < 6 months are generally treated with antibiotics because of overall risk of bacteremia and suppurative disease.

 

2.         Describe the physical exam findings associated with dehydration.  Discuss re-hydration.

Anonymous

In mild dehydration, there is some increased thirst with no abnormal findings on PE.  When fluid deficits reach 50 ml/kg in moderate dehydration, 5% to 10% loss in body wt, there is moderate thirst, dry mucous membranes, irritability, lethargy, decreased urine output, tenting of the skin, and tachycardia.  Tachycardia may also be a manifestation of fever or infection.  Skin turgor may be poor as well—where the skin remains “tented” rather than rapidly retract to its normal position.  As dehydration continues, acidosis and tachypnea contribute to poor peripheral perfusion.  In severe dehydration, c greater than 10% loss in body wt, signs of shock may appear with a rapid thready pulse, hypotension, and cool extremities.  There is intense thirst; irritability or lethargy with altered mental status, ad sunken eyes.  Most physical finding result from decreased extracellular fluid, PE findings are greatest in hyponatremic dehydration.  These pts may also have CNS symptoms with lethargy and seizures if the serum Na falls below 120 mEq/liter.  In contrast, hypernatremic dehydration is primarily a loss of intracellular fluid; therefore, PE finding may not be as marked b/c of the relative maintenance of circulating blood volume.  Shock is a late finding, and these infants may be lethargic but when stimulated become irritable with a shrill cry, hyperreflexive with increased muscle tone and may have seizures.

Anonymous Dershewitz 909-910

Physical exam- infants and children who are dehydrated will often appear ill.  Children should be weighed and vital signs obtained in all children.  Hyperpnea and tachypnea suggests decreased tissue perfusion resulting in metabolic acidosis.  Decreased circulating blood volume causes tachycardia, which can result in heart rates over 200 beats per minute in infants.  The extent of dehydration can be evaluated by examining the child's mental status, mucous membranes, skin turgor, peripheral perfusion, and peripheral pulses.  A careful examination of the abdomen should be performed to exclude signs of surgical diseases, such as bowel obstruction, appendicitis, and pyloric stenosis.

Oral rehydration therapy (ORT) with oral rehydration solutions is the treatment of choice for dehydration.  Patients with mild to moderate dehydration may take ORS over 4hrs or until stable.  Severe dehydration with evidence of shock or hemodynamic instability should receive IV or interosseous fluids immediately.

L.H. Emer. Med. Pg.840,874-76 and current peds pg 1284.

Page 9: Acute Gastroenteritis

Physical exam findings are related to the degree of dehydration, moderate, mild, or severe.

 

Finding                                                 mild                                                 moderate                                                       severe

decrease body wt        3-5%                          6-10%                             11-15%

BP                              normal                          normal                       normal-reduced

Tears                          decreased                    decreased                        absent                                                                                                                             

HR                              normal                        increased                       tachycardia

Skin turgor                  normal                        decreased                      decreased

Fontanelle                   normal                        sunken                           sunken

Mucous membrane     slightly dry                    dry                         mottled or gray; parched

Eyes                            normal                      sunken orbits                deeply sunken orbits

Cap refill                     2-3 sec                        3-4 sec                          more than 4 sec

Mental status              normal                     normal to listless            normal to lethargic

                                                                                                               or comatose

Urine output           mild oligouria                oligouria                          anuria         

 

 

Management of dehydrated children depends on the degree of fluid loss, as well as their ability  to tolerate oral liquids.  Mildly dehydrated patients who can tolerate pedialyte can usually be discharged home on clear liquids, with close follow up.  Moderately dehydrated patients generally require iv therapy, but oral rehydration is an option.  Oral rehydration for mild dehydration is 50 mL/kg over 4 hours.  For moderate dehydration, 100mL/kg over 4 hours.  Severely dehydrated patients require aggressive resuscitation.  Boluses of 20 mL/kg of normal saline are given until improved mental status, vital signs, and peripheral perfusion indicate stable intravascular volume.  In extreme situations, intraosseous line may be necessary.  Fluid replacement then consists of replacing 50% of the estimated volume defecit in the first 8 hrs and the remainder of the defecit in the next 16 hrs.  If diarrhea continues, maintenance fluids are added to the defecit replacement.

Page 10: Acute Gastroenteritis

D. Higbee, Tintinalli pg. 841

Ill appearance, capillary refill longer than 3 seconds, dry mucus membranes, absent tears.

Rehydrate with Oral Rehydration Solution (ORS), IV fluids.

 

3.         Given a case scenario, provide a differential diagnosis, identify most likely diagnosis, and form a treatment plan for a child presenting with gastroenteritis or dehydration.

Anonymous

This is a fairly broad area. I’ve listed the most common things that go wrong which are associated w/ abd. pain, N/V + diarrhea. Just remember to consider any associated  s/s, + hx of course.

  Gastroenteritis Dehydration

Differential Dx (causal)

-intestinal infxn:viral(rotavirus, most common), bacteria + parasites;

-appendicitis (w/o diarrhea);

-food intolerance(no fever + otherwise healthy);

-OM, pneumonia, UTI(w/ fever)

fever, vomiting, diarrhea, inadequate fluid intake, environmental (heat, burns), hyperventilation (metabolic acidosis)

Most Likely Dx

2nd to intestinal infxn 2nd to acute febrile illnesses + enteral infections

Tx supportive/ rehydration, feeding w/ appropriate diet

 

specific/ antibiotics

 

non-specific/ antidiarrheals,

after addressing the primmary cause(s) :rehydrate

L.H. Emerg. Med. Pg 841

Page 11: Acute Gastroenteritis

Causes of vomiting and diarrhea:

Viral

Bacterial

Noninfectious differential diagnosis include:  intestinal obstruction, appendicitis, toxic ingestion, metabolic disorder, head injury, other infections like UTI, hepatitis, etc.

 

Scenario’s

 

If a child is febrile, has abrupt onset of diarrhea, occurring more than 4 times per day or blood is in the stool, the illness is more likely to be caused by a bacterial pathogen and stool cultures are indicated.

 

Children that have been on antibiotics and develop bloody diarrhea should be evaluated with anaerobic stool cultures and assay of stool for C. difficile.

 

A history of biking or camping should prompt exam for ova and parasites.  In addition, a swab of mucous or bloody exudate from the stool should be placed in transport medium and sent to the lab for culture of Shigella.

 

In cases of persistent or recurrent diarrhea, especially with wt. loss or day care center exposure or in immunocompromised  children, stool samples should be collected in fixative and examined for G. lamblia, Entamoeba histolytica, and Cryptospordium.  Depending on geographic location or travel history, serologic testing for confirming E. hystolytica infection may be indicated.

 

See table 122-4 on page 843 of Emer. Med book for details of individual infections.

Also table 64.2 on pg 413 of primary care medicine book.

D. Higbee, Tintinalli pg. 841

See chart on page 841 in Tintinalli

Page 12: Acute Gastroenteritis

Anonymous p. 841-843, Tintinalli

            Differential Diagnosis:

            Viral cause, bacterial cause, parasitic causes, intestinal obstruction, appendicitis, toxic ingestion, metabolic disorder, head injury, and other infections.

 

            Most likely Diagnosis:

            Viral infection- specifically rotavirus

 

            Treatment:

            Oral rehydration fluid in all cases.  In cases of bacterial and or parasitic infection, add and an antibiotic or antiparasitic medication as appropriate.

 

4.         Discuss the difference between acute and chronic diarrhea.

Anonymous Dershewitz, pp. 582 & 590.

Acute diarrhea is usually a self-limited illness lasting a few days to a week, whereas persistent diarrhea typically persists longer than 2 weeks and may be associated with malabsorption, malnutrition, or both.  Almost all acute diarrhea in children is caused by intestinal infections of food intolerance, although diarrhea can also be secondary to an infection outside the bowel.

 In differentiating acute from chronic diarrhea, malasbsorption should be considered.  Clinical signs of gastrointestinal tract dysfunction may include chronic diarrhea; the passage of frequent, large, pale , oily, and foul-smelling stools; abdominal distention; or possibly increased appetite.

Anonymous Tintinalli

            Diarrhea lasting <3 weeks is acute, with diarrhea >3 weeks being chronic.  The acute diarrheas are of the greatest concern, for they are more apt to be a manifestation of an immediately life-threatening illness (infection, ischemia, intoxication, or inflammation).

L.H. Primary care med pgs 409-418

Diarrhea is categorized as acute if its duration is less than two weeks.  Four weeks or more or a pattern of recurrent diarrhea constitutes chronic diarrhea.  Acute diarrheas are usually self limited and non specific measures aimed at symptomatic relief are appropriate.  Chronic diarrhea

Page 13: Acute Gastroenteritis

requires an etiologic diagnosis and specific therapy.  Suppressing symptoms without identifying a cause may delay identification of a serious underlying condition.

D. Higbee, Tintinalli pg. 570

Acute diarrhea < 3 weeks

Chronic diarrhea > 3 weeks.

 

Acute diarrheas are of greatest concern, for they are more apt to be a manifestation of immediately life-threatening illness (infection, ischemia, intoxication, or inflammation).

Anonymous

o Acute Diarrhea:

o Diarrhea lasting <3 weeks is acute, with diarrhea.  The acute diarrheas are of the greatest concern, for they are more apt to be a manifestation of an immediately life-threatening illness (infection, ischemia, intoxication, or inflammation).

 

o Chronic Diarrhea: o >3 weeks being chronic

 

5.         Discuss incidence, symptoms, clinical presentation, physical exam findings, differential diagnosis, laboratory findings, and treatment of a child presenting with the following:

appendicitis

volvulus

                        intussusception

pyloric stenosis

Anonymous Dershewitz 575-577

Appendicitis

Page 14: Acute Gastroenteritis

Incidence:  the most common dz requiring surgery in childhood, estimated between 7%-12% of the general population, most common in teens and young adults, less common in children younger than 2 (less than 1% of all cases), rare in infants less than 1 year.

Symptoms: anorexia, vomiting, irritability, fever, discomfort on movement evolving over 12 hours  

Clinical Presentation:  peri-umbilical crampy pain progressing to LRQ pain if the inflamed appendix is retroceccal there may be no abdominal wall tenderness

Physical exam findings:   possible psoas irritation rectal or pelvic exam may reveal peritonitis or localized tenderness,  there may be abdominal guarding or rigidity, fever   repeated exam in 4-6 hours is recommended to rule out progressive disease.

Differential Diagnosis:  perforated viscus, obstruction, ectopic preg.  Gastroenteritis, UTI, mesenteric adenitis, intussusception, right lower lobe pneumonia

Laboratory findings:  Polymorphonuclear blood cell count is commonly elevated in appendicitis, abdominal ultrasound may be useful

Treatment:  prolonged observation and repetition of laboratory studies serve only to increase the risk of perforation, morbidity assoc with a perforated appy is high, referral to a pediatric surgeon should be made when an acute abdomen is suspected.   

 

volvulus: intermittent or persistent bile-stained vomiting after feedings, abdominal distention limited  initially to the epigastrium because only the stomache and duodenum are distended (the degree of distention depends on the pressure of swallowed air and degree of obstruction). Dehydration and electrolyte imbalance occur rapidly, fever, pain, scanty stools, diarrhea, and bloody stools are associated withprogressive volvus.  McCance p. 1384

 

intussusception: severe colicky pain that is paroxysmal and sometimes manifested in the very young pt only by screaming alternating with quiescent periods, with or without vomiting, is intussusception until proven otherwise. The pain is usually periumbilical and more severe than gastroenteritis. CLASSIC: RUQ mass, and current jelly stools (late finding). Early KUB and upright abdominal films may be normal. Dershewitz p. 575)

 

pyloric stenosis: 2-3 weeks after birth, an infant who has fed well and gains weight begins to vomit for no apparent reason -> vomiting gradually becomes more forceful usually occurring immediately after eating (may shoot 3-4 feet, often regurgitated through the nose). Vomitus consists of the bulk of the feeding + other feedings and contains no bile. Infants are hungry and

Page 15: Acute Gastroenteritis

irritable and want to eat again after vomiting. Prolonged food retention is characteristic and food may remain in infant’s stomach for 4 hours unless vomiting has occurred.  Constipation is the rule d/t little food reaching the intestine. If untreated: severe fluid & electrolyte imabalance, chronic malnutrition that can de fatal within 4-6 weeks. McCance pp.1383-1384

Anonymous dershewitz

Appendicitis- the most common disease requiring surgery in childhood.  The classic presentation is periumbilical crampy pain progressing to constant right lower quadrant pain in a child with fever, anorexia, vomiting, and leukocytosis. An infant with appendicitis man present with only irritability, discomfort on movement, and flexed hips, and if his inflamed appendix is retrocecal, there may be no abdominal wall tenderness. It may hurt to cry so the child may be silent even though or she is in pain.  Appendicitis must be considered in any ill looking child with changing abdominal examination and no obvious alternative cause. Differential diagnosis are gastroenteritis, urinary tract infection, mesenteric adenitis, pyelonephritis, and renal calculi. Primary lab finding is leukocytosis. Labs are CBC with differential and UA. A surgical exploration is indicated in any child with an acute abdomen and progressive signs of deterioration for which there's no satisfactory explanation.  In children with suspected appendicitis, prolonged observation and repetition of laboratory studies serve only to increase the risk of perforation.

 

Volvulus (Tnitinalli 847)– volvulus is a major life-threatening complication of malrotation.  It is the most urgent the GI emergencies in infants and children because of consequence of gangrene of the total midgut.  The time interval from the first symptom to the development of total midgut gangrene may be only a few hours.   The presenting symptoms are usually vomiting (becoming bilious) with or without abdominal distention, and streaks of blood in the stool.  Infants with symptoms of obstruction or bilious vomiting must receive prompt surgical consultation and active resuscitation. The most dramatic presentation in newborns is the sudden onset of an acute abdomen and shock, with a rigid and discolored abdomen associated with bilious or bloody vomiting and bloody stools, indicating the presence of gangrenous bowel.  On physical examination, such infants may appear pale and have grunting respirations, and approximately one-third of the infants will appear jaundiced. The vast majority of cases will appear in the first month of life.  The child should have flat and upright abdominal x-rays.  Occasionally an upper GI examination may reveal an abnormal location of the ligament of Treitz. D.D. = intussusception, duodenal stenosis, atresia, tummy ache. Surgical consult, admission, and immediate IV fluids are necessary including a nasogastric tube. Blood should be typed and Cross matched.  A blood cell count may identify early gut necrosis.  Electrolytes and the venous blood gases may identify sodium or potassium abnormalities or ongoing acidosis. Any child with vomiting or bloody stools who is identified as having an incompletely rotated bowel requires urgent laparotomy to prevent the development of midgut volvulus and total midgut gangrene.

 

Page 16: Acute Gastroenteritis

Intussusception - occurs when a portion of the alimentary tract is telescoped into another segment. It is the most common cause of intestinal obstruction between the three months ended six years of age and is rare under three months of age.  The male/ female ratio is 4:1. The classic patient is a robust 6 -18  month-old infant without prior difficulty. The youngster may be playing quietly in the playpen and suddenly stop playing, begins to cry, and even roll around in discomfort. This may resolve and the child seems fine with episodes recurring at more frequent intervals. Some children become very still, listless, and pale, and appear to be in a shock like a state to do to the visceral pain.  Vomiting is rare in the first few hours but usually develops after 6 to 12 hours. A classic sign is the “currant jelly” stool and is a late manifestation. Apathy or lethargy may be the only presenting sign of intussusception.  This can result in the infants receiving a lumbar puncture and other diagnostic studies, thus delaying diagnosis and treatment. Examination between attacks may reveal the often-described sausage-shaped tumor mass of intussuscepted bowel in the right side of the abdomen. If this mass is felt in the epigastrium, the long axis is usually horizontal.  One third of patients may not have a palpable mass.  Less typical presentations may suggest an intestinal obstruction.

            Diagnosis is based on history.  The apparent well-being of a child should not mislead the provider.  X-ray may show a mass or filling defect in the right upper quadrant of the abdomen.  Even in the presence of normal plain x-ray films, the history described demands a barium enema examination which demonstrates the classic “coiled spring. If it is obtained in the first 12 to 24 hours 80 percent of cases may be corrected by barium enema alone (5-10%recurrence rate).  If not a surgical intervention is indicated. A barium enema reduction is contraindicated if there   is free peritoneal air on plain films or if the infant has signs of peritonitis or sepsis. IV fluids  and antibiotics are essential in such cases.

           

pyloric stenosis - the infant with a history of non-bilious projectile vomiting must be considered to have pyloric stenosis. A familial incidence is noted in 50 percent of patients.  It is caused by diffuse hypertrophy and hypoplasia of the smooth muscle that narrows the antrum of the stomach to a small channel that can be easily obstructed. Usually occurs in the second or third week of life.  It seldom develops after the third month of life.  Initially, the infant may only regurgitate small amounts of milk, making it difficult to distinguish the cause of vomiting from simple regurgitation, gastric reflux, or milk intolerance.  Vomiting usually becomes projectile of after one week of symptoms and is never bile although it may have streaks of blood. Vomiting occurs just after one near the end of feeding, and afterward the infants will refeed hungrily unless the child has become malnourished or dehydrated. Physical examination usually demonstrates a hungry infant who has failed to gain weight over the past several weeks or has lost weight.  If one undresses and then feeds the infant, peristaltic waves can sometimes be seen passing left to right across the upper abdomen, just prior to an episode of vomiting. Palpation of a pyloric tumor – “the olive”- is pathognomonic. If no olive is palpated ultrasound is indicated. If negative but you are still suspicious an upper GI series can be performed. Once the diagnosis of pyloric stenosis has been confirmed or is highly suspected, surgical consultation should be obtained. Extensive and protracted vomiting in pyloric stenosis may lead to hypokalemia and hyponatremia. Replenish fluids with 5 percent dextrose in normal saline.

Page 17: Acute Gastroenteritis

L.H. Current Peds pg. 625-6

Appendicitis—most common indication for emergency abdominal surgery in childhood.

Frequency—increases with age and peaks between 15 and 30. Incidence of perforation is high—40%

Symptoms—fever, periumbilical abdominal pain (which then localizes to the RLQ)), anorexia, nausea, vomiting, constipation, diarrhea.

Clinical presentation—often atypical and includes generalized pain, tenderness around the umbilicus, and no leukocytosis.

Physical exam—Rectal exam should always be done and may reveal localized mass or tenderness.  Physical exam finding are often inconclusive, and many symptoms mimick appendicitis, so repeat abdominal exams should be done.

Lab findings—WBC count rarely higher than 15,000.  Pyuria, fecal leukocytes,and guiac positive stool are occasionally found.

Differential diagnosis—presence of intrathoracic infection, such as pneumonia, UTI,  acute GE, Crohns disease, ulcerative colitis, peptic ulcer, and other Medical and surgical conditions that lead to acute abdomens.

Treatment—Exploratory laparotomy or laparoscopy is indicated when the diagnosis of appy cannot be ruled out after a period of close observation. Post-op antibiotics is given to patients with gangrenous or perforated appy’s. A single intraoperative dose of cefotetan is given to all patients to prevent post-op complications.  

 

Intussusception—most frequent cause of intestinal obstruction in first 2 yrs of life. Three times more common in males than females.  85% of cases, the cause is not apparent.  Predisposing factors are:  polyps, Meckel diverticulum, Henoch-Schonlein purpura, lymphomas, lipomas, parasites, foreign bodies, adenovirus or rotovirus infections with hypertrophy of Peyer patches.  Occurs in patients with celiac disease and cystic fibrosis—related to the bulk of the stool in the small intestine.  Children over 6, lymphoma is the most commoncause.  Usually starts just proximal to the ileocecal valve, so that invagination is ileocolic.  Other forms:  ileoileal, and colocolic.  Swelling, hemorrhage, incarceration with necrosis of the intussuscepted bowel, and eventual perforation and peritonitis occur as a result of impairment of venous return.           

Symptoms:  paroxysmal abdominal pain with screaming and drawing up of knees, vomiting, diarrhea, bloody bowel movements with mucous, fever.  Abdomen is tender and becomes distended.  Sausage shaped mass may be felt in upper mid abdomen.  Some patients become lethargic between spasms or have seizures.  Intussusception may persist for several days if obstruction is not complete.

Page 18: Acute Gastroenteritis

Treatment:  Conservative:  barium enema is diagnostic and therapeutic.  It should not be done if signs of strangulated bowel, perforation, or severe toxicity are present.  Air insufflation of the colon under fleuroscopy is a safe alternative.  Surgical:  Surgery is required in 25% of cases, when bowel is perforated, or hydrostatic or pneumatic reduction is unsuccessful.

 

Current peds Pg 617

Pyloric Stenosis:  Incidence 1-8:1000 births.  4:1 male predominance.  Erythromycin therapy may be assoc. with development in infants under 30 days.

Symptoms:  vomiting begins within 2-4 wks and becomes projectile after every feeding.  Vomitus is blood-streaked.  Constipation, dehydration, wt loss, fretfulness, and apathy occur.  Upper abdomen may be distended after feeding, and prominent gastric peristaltic waves from left to right may be seen.  Deep palpation of Rt upper abdomen may reveal an olive sized mass.

Lab findings:  Hypochloremic alkalosis with potassium depletion, elevated Hemoglobin and hematocrit, elevated unconjugated bilirubin (2-5% of cases). Upper GI series shows delay in gastric emptying and an elongated narrowed pyloric channel with a double tract of  barium.  Enlarged pyloric muscle causes characteristic semilunar impressions on the gastric antrum.  Ultrasound shows a hypoechoic ring with a hyperdense center.  Thickness of circular muscle is more than 4mm in pyloric stenosis.

Treatment:  pyloromyotomy is treatment of choice.  Done laprascopically.  Prior to surgery you must repair hydration and electrolyte abnormalities.  Outlook is excellent following surgery.

 

emerg. Med.pg847-8

Volvulus—major life threatening complication of malrotation.  Complications occur most commonly in the first years of life, but can happen at any time.  Most urgent of GI emergencies in infants and children because of consequent gangrene of the total midgut.  Time from first symptom to total midgut gangrene may be only a few hrs.

Symptoms:  vomiting, with or without abdominal distention, streaks of blood in stool.  Most dramatic presentation in newborns is sudden onset of acute abdomen and shock, with a rigid and discolored abdomen assoc. with bilious or bloody vomiting and bloody stools, indicating presence of gangrenous bowel.

Physical exam:  child appears pale with grunting respirations, 1/3 of pts will be jaundiced.  Older children, pain is constant, not colicky.  Flat and upright abdominal xrays show the presence of a loop of bowel overriding the liver.  An upper GI exam may reveal an abnormal location of the ligament of Treitz.

Page 19: Acute Gastroenteritis

Differential diagnosis:  intussusception, duodenal stenosis, and atresia can produce a clinical picture similar to midgut volvulus.  

Treatment:  Prompt surgical consultation, active resuscitation, and hospital admission.  IV fluids immediately and place a nasogastric tube.  Type and cross blood.  WBC count may identify early gut necrosis, electrolytes and venous blood gases may identify sodium or potassium abnormalities or ongoing acidosis.  Any child with vomiting and bloody stools who is identified as having an incompletely rotated  bowel requires urgent laparotomy to prevent development of midgut volvulus and total midgut gangrene.

D, Higbee Tintinalli pg 847-850

Appendicitis: Although appendicitis can occur in children younger than age 2, the presentation is usually one peritonitis or sepsis.  Over age 2, appendicitis becomes a more important part of differential diagnosis of abdominal pain.  Symptoms:  Progression of sx’s, anorexia, mild-mod periumbilical pain, vomiting, then the movement of pain to RLQ.  PE:  Limited motion of lower abdomen, possibly abdominal distention, guarding and rebound tenderness, if perforated, a RLQ mass. Clinical presentation: Pain, guarding, tenderness, fever.  Differential Diagnosis: Peritonitis, sepsis, gastroenteritis.  Lab Findings: Elevated WBC, X-ray reveals an appendicolith.  Treatment: Surgical consult, admit to hospital, nothing by mouth (NPO), IV fluids, rectal acetaminophen.  

 

Volvulus: Complications occur most commonly in the first year of life.  Symptoms: Vomiting, (ultimately becoming bilious), with or without abdominal distention.  The most dramatic presentation in newborns is sudden onset of an acute abdomen and shock, with a rigid and discolored abdomen associated with bilious or bloody vomiting and bloody stools, indicating the presence of a gangrenous bowel.  Clinical Presentation/PE:  Such infants may appear pale and have grunting respirations, and approximately 1/3 of the infants will appear jaundiced.  Differential Diagnosis: Intussusception, duodenal stenosis, or atresia.  Lab findings: Flat and upright abdominal x-rays, the presence of a loop of bowel overriding the liver is suggestive of diagnosis.  Occasionally, an upper GI exam may reveal an abnormal location of the ligament of Treitz.  Treatment: Prompt surgical consultation, active resuscitation, hospital admit.  IV fluids, NG tube placed.  Blood typed and cross-matched.  Labs: Electrolytes and venous blood gases may identify sodium or potassium abnormalities or ongoing acidosis.

 

Intussusception:  Occurs when a portion of the alimentary tract is telescoped into another segment.  It is the most common cause of intestinal obstruction between 3 mo and 6 years of age and is rare under 3 mo.  Male/female ratio is 4:1.  Symptoms: Sudden onset of pain.  Episodes may recur at more frequent intervals with the duration of the painful attacks increasing.  Clinical Presentation: Apathy and lethargy.  PE: Sausage-shaped tumor mass of intussuscepted bowel in right side of abdomen.  Differential Diagnosis: Volvulus.  Labs: x-ray may show a mass or filling defect in RUQ, barium enema demonstrates the classic “coiled spring”.  Treatment:

Page 20: Acute Gastroenteritis

Barium enema may resolve problem if caught in the first 12 to 24 hrs of the developing intussusception.  If not resolved, surgical intervention is indicated.  

 

Pyloric Stenosis:  Caused by diffuse hypertrophy and hypoplasia of the smooth muscle that narrows the antrum of the stomach to a small channel that can be easily obstructed.  Affects approximately 1 in 150 male and 1 in 750 female patients.  Symptoms/Clinical presentation: Initially the infant may only regurgitate small amounts of milk, gastric reflux, or milk intolerance.  Vomiting usually becomes projectile within a week of onset of symptoms, and vomitus is never bile stained, although it may have streaks of blood.  Vomiting occurs just after or near the end of feeding, and afterward the infant will feed hungrily.  Constipation noted.  PE: Hungry infant that has failed to gain weight for several weeks.  Jaundice occurs in 1-2% of cases.  Peristaltic waves can sometimes be seen passing left to right across upper abdomen.  Palpations of pyloric tumor-the “olive”-is pathognomic.  In advanced cases, PE shows dehydration and lethargy.  Child may appear moribund, with sunken eyes.  Decreased elasticity of skin and loss of subcutaneous tissues.  Palpation of the “olive” confirms diagnosis.  Labs: Ultrasonography.  If normal, upper GI series can be done.  Treatment: Surgical consult, oral intake is restricted, IV started.  Dehydration and electrolyte abnormalities must be corrected before surgery.  

Anonymous

            Appendicitis p. 625-626, Current Pediatrics

Incidence:  frequency increases with age and peaks between 15 and 30 years.

 

Symptoms:  fever and periumbilical pain which localizes to the RLQ accompanied by signs of peritoneal irritation.  Anorexia, vomiting, constipation, and diarrhea also occur.

 

Physical Exam:  Rectal exam may reveal localized mass or tenderness, tenderness on palpation of RLQ, rebound tenderness in RLQ, and referred rebound tenderness to RLQ when LLQ is palpated.

 

Differential Diagnosis:  intrathoracic infection, UTI, foreign body, gallstone, parasite infection, tumor, hernia. Volvulus, pyloric stenosis, Crohn’s disease, ulcerative colitis, peptic ulcer, acute gastroenteritis, pseudomembranous enterocolitis, sepsis, peritonitis, pyelonephritis, pancreatitis, renal stones, PID, abdominal trauma, Lead poisoning, sickle cell crisis, Familial Mediterranean fever, dialectic, acidosis, torsion of the testis, torsion of ovarian pedicle.

Page 21: Acute Gastroenteritis

 

Lab findings:  WBCs are seldom higher than 15,000/microL, pyuria, fecal leukocytes, and guaiac positive stool.

 

Treatment:  Exploratory laparotomy or laparoscopy is indicated when the diagnosis of appendicitis cannot be ruled out after a period of close observation.  Post-op antibiotics are given to patient with gangrenous or perforated appendix.  A single intraoperative dose of cefotetan is given to all patients to prevent post-op complications.

 

            Volvulus p.620-621 Current Pediatrics; Tintinalli 847-848

Incidence:  most urgent of GI emergencies in infants and children and accounts for 10% of neonatal intestinal obstructions.

 

Physical Exam: presents with recurrent bile stained vomiting or acute bowel obstruction.  May present with ascites and later in life there may be signs of intermittent intestinal obstruction, malabsorption, protein-losing enteropathy, or diarrhea.  Associated with congenital anomalies, especially cardiac.

 

Differential Diagnosis: intussusception, duodenal stenosis, atresia.

 

Lab findings: WBC, electrolytes, venous blood gas, upper GI series shows duodenojejunal junction on the R. side of the spine along with jejunal loops.  Diagnosis of malrotation can be further confirmed by barium enema.

 

Treatment: prompt surgical consult, active resuscitation, and hospital admission.  IV fluid must be started immediately and a nasogastric tube placed.

 

            Intussusception p. 627 Current Pediatrics

Page 22: Acute Gastroenteritis

Incidence:  most frequent cause of intestinal obstruction in first 2 yrs. of life.  Three times more common in males than females.  85% of cases, the cause is not apparent.  Predisposing factors are:  polyps, Meckel diverticulum, Henoch-Schönlein purpura, lymphomas, lipomas, parasites, foreign bodies, adenovirus or rotavirus infections with hypertrophy of Peyer patches.  Occurs in patients with celiac disease and cystic fibrosis—related to the bulk of the stool in the small intestine.  Children over 6, lymphoma is the most common cause.  Usually starts just proximal to the ileocecal valve, so that invagination is ileocolic.  Other forms:  ileoileal, and colocolic.  Swelling, hemorrhage, incarceration with necrosis of the intussuscepted bowel, and eventual perforation and peritonitis occur as a result of impairment of venous return.

 

Symptoms: paroxysmal abdominal pain with screaming and drawing up of knees, vomiting, diarrhea, bloody bowel movements with mucous, fever.  Abdomen is tender and becomes distended.  Sausage shaped mass may be felt in upper mid abdomen.  Some patients become lethargic between spasms or have seizures.  Intussusception may persist for several days if obstruction is not complete.

 

Physical exam: on physical exam a sausage-shaped mass maybe found usually in the upper mid abdomen.  Some patients show signs of altered consciousness – particularly lethargy between spasms of pain – or have seizures.

 

Differential Diagnosis: volvulus, duodenal stenosis, atresia.

 

Lab findings: barium enema is diagnostic, x-ray examination of the abdomen.

 

Treatment: reduction by barium enema, hydrostatic reduction, or air insufflation.  In extremely ill patients when other methods of reduction have failed surgery is required.

 

            Pyloric stenosis p. 617 Current Pediatrics

Incidence: The cause is postnatal circular muscular hypertrophy leading to gastric outlet obstruction.  The incidence is 1-8:100 births with a 4:1 male predominance.  There is a positive family history in 13 % of the cases.

 

Page 23: Acute Gastroenteritis

Symptoms: vomiting begins between 2 and 4 weeks and rapidly becomes projectile after every feeding.  Onset of symptoms may be delayed in premature babies.  Vomit may be blood streaked.  The infant is hungry and nurses avidly.  Constipation, dehydration wt. loss, fretfulness, and finally apathy occur.  Upper abdomen may be distended after feeding and prominent gastric peristaltic waves from L. to R. may be seen.  

 

Physical exam: an olive sized mass may be felt on deep palpation on the RUQ.

 

Differential Diagnosis: volvulus, intussusception, atresia.

 

Lab findings: hypochloremic alkalosis with potassium depletion, ↑ Hg and ↑ Hct, and ↑ unconjugated bilirubin in 2%-5% of the cases.

 

Treatment: pyloromyotomy is the treatment of choice and is performed laparoscopically.

  

6.         Given a case scenario provide a differential diagnosis, identify most likely diagnosis, and treatment for a child presenting with abdominal pain.

Anonymous Dershowitz, pp 574-581

DDx for acute abdominal pain: appendicitis, perforated viscus, bowel obstruction, ruptured ectopic pregnancy, gastroenteritis, UTIs, mesenteric adenitis, intussusception, Meckel’s diverticulum, PUD, biliary tract disease, Yersinia enterocolits, pancreatitis, primary peritonitis (usually in children with other chronic disease), non-GI causes such as pneumonias, strep pharyngitis, tonsillitis, renal/ureteral calculi, testicular tortion, diabetic ketoacidosis, PID, pelvic endometriosis, corpus luteal hematoma, ovarian cyst, chronic salpingitis and mittelschmerz.

DDx for recurrent abdominal pain: Most do not have an organic cause (psychosomatic). Organic causes include urinary tract disease (including intermittent ureteropelvic junction obstruction), H. Pylori infection, PUD, lactose or sorbitol intolerance, giardiasis, early Crohn’s disease, Henoch-Schonlein purpura, celiac disease, Yersinia enterocolits, pancreatitis, gallstone disease, lead poisoning, abdominal epilepsy, osteomyelitis of the spine, psoas abscess and hernias (abdominal, inguinal or femoral). In adolescence, consider pelvic disease in females, always get sexual history because adolescents with sexual concerns may present with non-organic abdominal pain.

Page 24: Acute Gastroenteritis

*This question essentially asks for a summary of the entire subject. I’ve included the differential, but learning to differentiate between diseases is kind of the point of the rest of the objectives.

Anonymous dershewitz

            Recurrent abdominal pain typically presents in children 5 to 10 years old, although the diagnosis can be considered in children between the ages of three and 16 years. Patients usually complained of a dull, colicky, periumbilical pain that is intermittent on a daily basis with a complete recovery between episodes. The pain is rarely focal and localization away from the umbilicus should suggest another diagnosis. The characterization of pain is usually described in vague terms, and no distinct association between the pain and eating, position, or time of day can be found. Abdominal pain does not awakened the patient of sleep, but it may interfere with the ability of the patient to fall asleep. Other symptoms are low grade fever, pallor, headache, and vomiting. It should be made clear early on that there is a likelihood of a finding an organic etiology and that there's no physical danger to the child.  The history must rule out suspicion of underlying organic illness and look for evidence of emotional stress or behavior problems.  The physical examination is usually unremarkable, but it must be thorough to rule out organic disease. Lab tests: CBC, sed rate, total protein and albumin, UA, stool guaiac, stool for ova and parasites. If indicated: lactose breath test, LFT’s, amylase, lipase, electrolytes, imaging.

 

 The first step in diagnosing recurrent abdominal pain is to verify that the symptoms are chronic.  An ill-looking child, or a child with persistent fever, weight loss, growth failure, or anemia suggests that recurrent abdominal pain is not the correct diagnosis. Abdominal tenderness, pain that weakens the child that night, bilious vomiting, bloody emesis, melena, suggest a gastrointestinal etiology. Urinary tract disease is the most common organic cause of chronic abdominal pain and must be considered even without the symptoms of dysuria and frequency. A positive diagnosis of non-organic abdominal pain can be made.  The discussions with the family should emphasize that recurrent abdominal pain is common and that many possible causes exist.  Counseling should be initiated to help the child and family deal with symptoms that may not immediately disappear.  DD = PUD, GERD, UTI, lactose intolerance,infection, intermittent small bowel obstruction.

Anonymous p. 641-642, Current Pediatrics

DIFF DX

Appendicitis

Enterocolitis

Congenital anomalies

Inborn errors of metabolism

Page 25: Acute Gastroenteritis

Cystic Fibrosis

Intussusception

Sickle cell anemia

 

Most likely

Appendicitis

 

TX:  

The physical examination is usually unremarkable, but it must be thorough to rule out organic disease. Lab tests: CBC, sed rate, total protein and albumin, UA, stool guaiac, stool for ova and parasites. If indicated: lactose breath test, LFT’s, amylase, lipase, electrolytes, imaging.  The treatment consists of reassurance and is based on a thorough physical appraisal and a sympathetic explanation of the functional nature of the complaint, the child and his or her parents should be strongly encouraged that regular activity should be resumed, especially school attendance.  Therapy for emotional problem is sometimes required, but drugs should be avoided.  In older patients, and in those with what appears to be visceral hyperalgesia, amitriptyline in low doses may occasionally be helpful.  Antispasmodic medications are rarely helpful and should be reserved for patients with more typical irritable bowel complaints.

 

7.         Hepatitis - Discuss pathophysiology, clinical presentation, work-up, treatment, and prevention.

Anonymous Dershewitz p.605-608

Pathophysiolgy:  Regardless of the mechanism leading to hepatocellular injury, one of the consequences is a loss of the selective permeability of the cellular membrane, with spilling of AST and ALT into the systemic circulation.  Impaired clearance of unconjugated bilirubin from serum occurs because of diminished uptake of bilirubin into hepatocytes, impaired intracellular binding, and decreased conjugation.  Unconjugated bilirubin then accumulates in the circulation, producing icterus.  The excretion of conjugated bilirubin is also altered and contributes to the icterus as well as causing dark, tea-colored urine.  Clay-colored or pale stools occur because of a decrease in or absence of pigments that originate from both diet and bile.

Clinical presentation:  the prodrome of acute viral hepatitis consists of malaise, fatigue, anorexia, nausea, vomiting, low-grade fever, and abdominal pain.  Coroli’s triad of headache, rash and arthralgias occurs in 5-15% of cases.  An icteric phase often follows the prodrome and

Page 26: Acute Gastroenteritis

is characterized by dark urine; pale, clay-colored stools; and jaundice.  With the onset of jaundice, some symptoms abate, whereas others worsen.  The fever, arthralgias, and headaches typically disappear.  Jaundice usually peaks within 5 to 10 days.   The first signs of recovery include the disappearance of nausea and vomiting with the return of appetite.  Malaise, usually the first symptom to appear, is also the last to resolve.

Work-up:  

Hx: A complete history to include possible exposure to hepatitis (contact with other persons infected, travel to a developing country, shellfish ingestion, ETOH, IV drugs, blood products, homosexual practice, hepatotoxic medications, day care attendance, etc.).

PE:  A thorough GI exam is in order, look for:  hepatomegaly and pain on percussion over the liver, ascites, spider hemangiomas, collateral circulation over the abdomen, and splenomegaly.  The skin may have a papular acrodermatitis with HBV.  The eye may reveal Kayser-Fleischer ring of Wilson’s disease.  Petechiae and mucosal bleeding reflect a coagulopathy.  The extremities may reveal clubbing in patients with long-standing disease.  With fulminant hepatitis the patient will manifest encephalopathy with asterixis and fetor hepaticus.

Labs:  LFTs (ALT, AST, bilirubin, GGT, alkaline phosphatase, and PT).  Also, you should look for specific viruses (HBV, HAV, HCV, etc.).

Treatment:  management of viral hepatitis is directed at minimizing its complications.  Control of coagulopathy and bleeding may be accomplished with Vitamin K, hypoglycemia with glucose infusion, ascites with sodium restriction, etc.

Prevention:  vaccines.

Anonymous

•        Pathophysiology- loss of selective permeability of cell membrane  and spilling of aspartate amino transferase and alanine aminotransferase into the systemic circulation. An impaired clearance of unconjugated bilirubin from serum occurs because of a diminished uptake of bilirubin into the hepatocyte, impaired intracellular binding, and decreased conjugation. Unconjugated bilirubin then accumulates in the circulation, producing icterus.

•        The prodrome of acute viral hepatitis consists of malaise,fatigue, anorexia, nausea, vomiting, low grade fever, and abdominal pain.

•        Workup includes a complete history, jaundice, and possible predisposing conditions for acquiring hepatitis. Lab tests included in the LST and AST, bilirubin, alkaline phosphatase, PT.

•        Therapy for viral hepatitis is directed at minimizing the complications.  Control of coagulopathy and bleeding may be obtained with the injection of vitamin K. and plasma infusions hypoglycemia is managed by constant glucose infusion and ascites by restricting

Page 27: Acute Gastroenteritis

salt intake.  A specific treatment should be applied when a metabolic, immunologic, or drug-induced cause of the hepatitis is identified.

•        Prevention is via vaccination. (A and B)

Michelle Current Pediatric 16th Edition Page 660

   Pathophysiology

    Hep A- Enterovirus (RNA)

    Hep B- Hepadnavirus (DNA)

    Hep C- Flavivirus (RNA)

    Hep D- Incomplete (RNA)

    Hep E-  Calicivirus (RNA)

 

            Clinical Presentation-page 661-

Hep A- Ask history questions to include direct exposure to a previously jaundiced individual, consumption of seafood or contaminated water, or recent travel to an area of epidemic infection.

 

            Symptoms and Signs-page 661-

Hep A- Fever, anorexia, vomiting, headache, and abd. pain are usually symptoms.  Darkening of the urine precedes jaundice, which peaks in 1-2 weeks and then begins to subside.  The stools become light or clay colored during this time.  Clinical improvement can occur as jaundice develops.  Tender hepatomegaly and jaundice are typically present.  Splenomegaly is variable.

 

Hep B- Slight fever, mild GI upset, visible jaundice is usually the first significant finding.  Darkening of the urine and pale or clay colored stools.  Hepatomegaly is present.

 

Hep C- Incubation of 5 months.  Asymptomatic despite development of chronic hepatitis.  Flu-like prodromal symptoms and jaundice occur in less than 25% of cases.  Hepatosplenomegaly may or may not be present.

Page 28: Acute Gastroenteritis

 

            Treatment-page 660

    Hep A- None

    Hep B- Interferon-alfa (or lamivudine in adults)

    Hep C- Interferon-alfa (plus ribavirin in adults), Pegylated interferon (adults).

    Hep D- Interferon-alfa

    Hep E- None

 

            Prevention

   Tell patients about good hand washing skills with soap.  Warn people the risks of unprotected sex.  Tell patients about using sterile needles.  Vaccination of children. 

Anonymous

Pathophysiology- loss of selective permeability of cell membrane and spilling of aspartate amino transferase and alanine aminotransferase into the systemic circulation. An impaired clearance of unconjugated bilirubin from serum occurs because of a diminished uptake of bilirubin into the hepatocyte, impaired intracellular binding, and decreased conjugation. Unconjugated bilirubin then accumulates in the circulation, producing icterus.

The prodrome of acute viral hepatitis consists of malaise, fatigue, anorexia, nausea, vomiting, low grade fever, and abdominal pain.

Workup includes a complete history, jaundice, and possible predisposing conditions for acquiring hepatitis. Lab tests included in the LST and AST, bilirubin, alkaline phosphatase, PT.

Therapy for viral hepatitis is directed at minimizing the complications.  Control of coagulopathy and bleeding may be obtained with the injection of vitamin K. and plasma infusions hypoglycemia is managed by constant glucose infusion and ascites by restricting salt intake.  A specific treatment should be applied when a metabolic, immunologic, or drug-induced cause of the hepatitis is identified.

Prevention is via vaccination. (A and B)

 

Page 29: Acute Gastroenteritis

8.         Vomiting - Discuss pathophysiology, clinical presentation, differential diagnosis (including type of vomiting and associated symptoms), work-up and management.

Anonymous

Pathophysiology – active process in which a strong contraction of the abdominal muscles leads to increased intra-abdominal pressure and elevation of the diaphragm into the thorax.

Clinical presentation – hx of wt loss or gain, presence or absence of bile, undigested food, mucus or blood in vomitus, relationship of vomiting to meals, early morning vomiting, vomiting after meals, fever, diarrhea, jaundice, dysphagia, head trauma or pain.

Differential DX

Infancy – overfeeding, GERD, soy/milk intolerance, sepsis, pyloric stenosis, intestinal obstruction, malrotation, subdural effusion, metabolic disorder, adrenal insufficiency, neurologic damage

Childhood ( in addition to those listed in infancy) – gastroenteritis, PUD, pneumonia, OM, DM, increased ICP, meningitis, appendicitis, hepatitis, Reye’s syndrome, intussusception, migraine, abdominal epilepsy, ketonic, hypoglycemia, drug intolerance/toxicity

Adolescence (in addition to all of those listed above) – Anorexia nervosa, bulimia, pregnancy

Work-up –

 Hx to include feeding patterns, quantity and quality of vomitus, child’s hunger, timing of vomiting, and concurrence of  abdominal pain, diarrhea, and dysphagia, fever, head trauma, drugs, headaches, polydipsia, polyuria, muscle weakness, wt loss.

PE – eval of each organ system, vital signs, assessment of hydration state

Lab tests – APT test, barium enema, upper GI, abdominal US, plain abdominal x-ray, CBC, urine pregnancy test, Chemistry profile, blood cultures, CXR, upper endoscopy, stool cultures.

Management – depends on DX  no standardized approach exists for the management of vomiting!

Anonymous

•        Patho – With vomiting, the pyloric sphincter contracts and the cardia rises and opens so that the gastric contents are ejected forcefully by the high intra-abdominal pressure. During the abdominal contraction, no active contractions are noted in the esophagus.  After several seconds, peristaltic activity starts in the upper esophagus clearing its contents into the stomach. The esophageal sphincter closes after this secondary wave.  Emesis is mediated by the vomiting center in the reticular brain formation. Causes of vomiting in infancy may be

Page 30: Acute Gastroenteritis

identified by seeking information about diets and previous medication that may be related to an intolerance of a specific food or drug. Information on the quality and quantity of the vomiting this should be obtained because it is useful in localizing the source of the problem.  The presence of bile suggests an obstruction distal to the duodenum, undigested food may be found in disorders affecting the esophagus, such as achalasia, or stricture. Mucus is often noted in infancy with feeding problems or pylorospasm.

•        Presentation – Early morning vomiting before eating is observed frequently in patients with intracranial hypertension ( if projectile) or in metabolic diseases.  Vomiting that occurs when a meal is finished suggests psychological disorders or peptic ulcer. Regurgitation is an effortless expulsion of the gastric contents, usually occurring in infancy.  Rumination is the regurgitation of ingested food, which is then rechewed and reswallowed. Wretching involves voluntary attempts to empty the stomach and uses the thoracic musculature.

•        Differential Dx – Patients may vomiting for several reasons.  These include inflammation of the mucosa of the gastrointestinal tract (gastroenteritis), stretching of an organ or membrane(bowel obstruction or otitis media), vestibular reflex (seasickness), stimulation of the chemoreceptor trigger zone (diabetes mellitus) and increased intracranial pressure(brain tumor).  

•        Work-Up – Since the vomiting as so many causes, the physical examination should be complete and should include an evaluation of each organ system, vital signs, and assessment of the state of hydration. Any child with emphasis plus colicky abdominal pain should be evaluated immediately.

•        Management – If the child can tolerate small sips of fluid an attempt should be made at oral rehydration.  An electrolytes solution is optimal but if this is not available a decarbonated soft drink, jello, or tea can be used for a limited period. Complications of vomiting that require admission are aspirations, GI bleeding, moderate to severe dehydration.  A young child with mild dehydration and continued vomiting may also warrant hospitalization.

Michelle Current Pediatric 16th Edition Page 614

    Clinical Presentation- Assessment of the child with recurrent episodes of vomiting should start with a complete history, physical examination and description of the vomitus.  Emesis of gastric contents is characteristic of gastric outlet obstruction, central nervous system masses or infection, peptic disease, urinary tract infection, otitis or sinusitis, metabolic disease (especially acidosis), rumination and psychogenic vomiting.  Gastroesophageal reflux should be suspected in a healthy child with effortless postprandial spitting.

      Work Up- A urinalysis and culture to rule out UTI or urinary obstruction.  Serum electrolytes should be obtained.  X-rays and scans to rule out increased ICP, or abnormal CNS, chest or sinuses is done specific to the exam.

  Treatment- should be done based on your findings of the work-up.

Page 31: Acute Gastroenteritis

Sung K, CMDT 2003 p.524, Current Pediatric p.614, Tintinalli p.838-42

a)  the medullary vomiting center can be stimulated by four different sources of afferent input:

     (1) Afferent vagal fibers (rich in serotonin receptors) and splanchnic fibers from the GI viscera; these may be stimulated by biliary or GI distention, mucosal or peritoneal irritation, or infection.

     (2) Fibers of the vestibular system which have high concentrations of histamine H1 and muscarinic cholinergic receptors

     (3) Higher CNS centers; certain sights, smells, or emotional experiences may induce vomiting.

     (4) The chemoreceptor trigger zone, located outside the blood-brain barrier in the area postrema of the medulla, which may be stimulated by drugs and chemotherapeutic agens, toxins, hypoxia, uremia, acidosis, and radiation therapy.

     Note: in children abnormal serum electrolytes (Na, K, Mg, Ca) and increased intracranial pressure may cause vomiting

b) Clinical presentation – dehydration

c)  Differential Dx

     (1) Gastric content vomit – gastric outlet obstruction, CNS mass or infection, peptic disease, UTI, otitis or sinusitis, metabolic diseases, rumination, psychogenic

     (2) Bilious vomit – small intestinal obstruction

     (3) Bloody vomit – swallowing maternal blood in newborns, oropharyngeal lesions, nosebleed, peptic disease, bleeding disorders, foreign bodies, and esophageal varices.

d) Radiograph for possible obstructions, UA and culture, chemistry panel of electrolytes.

e)  Tx with oral hydration or IV hydration

Anonymous

Pathophysiology – active process in which a strong contraction of the abdominal muscles leads to increased intra-abdominal pressure and elevation of the diaphragm into the thorax.

Clinical presentation – hx of wt loss or gain, presence or absence of bile, undigested food, mucus or blood in vomitus, relationship of vomiting to meals, early morning vomiting, vomiting after meals, fever, diarrhea, jaundice, dysphagia, head trauma or pain.

Page 32: Acute Gastroenteritis

Differential DX

Infancy – overfeeding, GERD, soy/milk intolerance, sepsis, pyloric stenosis, intestinal obstruction, malrotation, subdural effusion, metabolic disorder, adrenal insufficiency, neurologic damage

Childhood ( in addition to those listed in infancy) – gastroenteritis, PUD, pneumonia, OM, DM, increased ICP, meningitis, appendicitis, hepatitis, Reye’s syndrome, intussusception, migraine, abdominal epilepsy, ketonic, hypoglycemia, drug intolerance/toxicity

Adolescence (in addition to all of those listed above) – Anorexia nervosa, bulimia, pregnancy

Work-up –

Hx to include feeding patterns, quantity and quality of vomitus, child’s hunger, timing of vomiting, and concurrence of  abdominal pain, diarrhea, and dysphagia, fever, head trauma, drugs, headaches, polydipsia, polyuria, muscle weakness, wt loss.

PE – eval of each organ system, vital signs, assessment of hydration state

Lab tests – APT test, barium enema, upper GI, abdominal US, plain abdominal x-ray, CBC, urine pregnancy test, Chemistry profile, blood cultures, CXR, upper endoscopy, stool cultures.

Management – depends on DX no standardized approach exists for the management of vomiting!

 

9.         Given a case scenario provide a differential diagnosis, identify most likely diagnosis, and treatment for a child presenting with vomiting.

Anonymous  Dershewitz p. 567 - 569

DDx:

Infant                                      Childhood                               Adolescence

(In addition to those in infants)   (In addition to those in Child)

Overfeeding                              Gastroenteritis                          Anorexia nervosa

Gastroesophageal reflux                        Peptic Ulcer disease                 Bulimia

Soy/Milk intolerance                 Pneumonia                                Pregnancy

Sepsis                                      Otitis media                             

Page 33: Acute Gastroenteritis

Pyloric stenosis                         Diabetes mellitus

Intestinal obstruction                 Increased Intracranial pressure

Malrotation                               Meningitis

Subdural effusion                      Appendicitis

Metabolic disorder                   Hepatitis

Adrenal insufficiency                 Reye’s syndrome

Neurologic damage                   Intussusception

                                                Migraine

                                                Abdominal epilepsy

                                                Ketotic hypoglycemia

                                                Drug intolerance / toxicity

 

Dx: it is especially important to identify other anatomic congenital or acquired causes of vomiting.

Treatment:

In general, no standardized approach exists for the mgmt of vomiting, because therapy depends on an identifiable etiology.

Any child w/ emesis plus colicky abdominal pain should be evaluated immediately.

Assess the level of dehydration; an attempt should be made at oral rehydration. A physiologic glucose-electrolyte solution is optimal for dehydration of infants w/ vomiting and diarrhea who are able to take fluids by mouth.

Corticosteriod therapy may be life saving if adrenal insufficiency is suspected.

Anonymous Derschewitz 567-569

Differential/ Causes of Vomiting:

Infancy Childhood Adolescence

Page 34: Acute Gastroenteritis

•        Overfeeding - Weight gain is a clue

•        Gastroesophageal reflux

•        Soy/milk intolerance - Vomiting in a healthy appearing infant

•        Sepsis

•        Pyloric stenosis - Undigested food in the vomitus

•        Intestinal obstruction - Bilious emesis in a newborn must be evaluated immediately; assume bowel obstruction until appropriate diagnostic studies have been performed

•        Abdominal distension

•        Malrotation

o       Intermittent vomiting

o       Bilious emesis in the newborn

o       Limited time of relative well-being before presenting in septic shock

•        Subdural effusion

•        Metabolic disorder

•        Adrenal insufficiency

•        Neurologic damage

Gastroenteritis - Often associated with diarrhea

Peptic ulcer disease - Vomiting occurring when a meal is finished

Pneumonia - fever

Otitis media - fever

Diabetes

Increased intracranial pressure

o Projectile vomiting

o Vomiting often occurs in the early morning before eating

Meningitis - fever

Hepatitis - jaundice

Reye’s syndrome - aspirin ingestion

Intussusception - intermittent or colicky abdominal pain

o palpable mass

Migraine - headache

o cyclic vomiting

Abdominal epilepsy - headache

o cyclic vomiting

Ketotic hypoglycemia

Drug intolerance/ toxicity

•        In addition to those listed for infancy and childhood

•        Anorexia Nervosa

•        Bulimia

•        Pregnancy

 

Most likely diagnosis - ???

Page 35: Acute Gastroenteritis

The book did not point out a “most likely diagnosis” in particular.  It does note that vomiting in a healthy appearing infant suggests dietary protein intolerance, but also notes that other etiologies must first be ruled out.  Most of these diagnoses would come to mind based on certain things in the history.  Similarly, the treatment varies for each. 

Sung K, Tintinalli p.838

        a)  Differential Diagnosis

            Infection – viral, bacterial, meningitis, sepsis, appendicitis, OM, UTI,

            Intestinal obstruction – pyloric stenosis

            Toxic ingestion – contaminated food

            Metabolic disorders – diabetic ketoacidosis, hyperammonemia

            Head injury/increased intracranial pressure

            Hepatitis

            Cholecystitis

            Pancreatitis

            Abdominal trauma

        b) Most likely diagnosis – self-limited viral disorder

        c)  Treatment – Rehydration

Anonymous

            Differential/ Causes of Vomiting:

Infancy Childhood Adolescence

•        Overfeeding - Weight gain is a clue

•        Gastroesophageal reflux

•        Soy/milk intolerance - Vomiting in a healthy appearing infant

Gastroenteritis - Often associated with diarrhea

Peptic ulcer disease - Vomiting occurring when a meal is finished

Pneumonia - fever

Otitis media - fever

 

Page 36: Acute Gastroenteritis

•        Sepsis

•        Pyloric stenosis - Undigested food in the vomitus

•        Intestinal obstruction - Bilious emesis in a newborn must be evaluated immediately; assume bowel obstruction until appropriate diagnostic studies have been performed

•        Abdominal distension

•        Malrotation

o       Intermittent vomiting

o       Bilious emesis in the newborn

o       Limited time of relative well-being before presenting in septic shock

•        Subdural effusion

•        Metabolic disorder

•        Adrenal insufficiency

•        Neurologic damage

Diabetes

Increased intracranial pressure

o Projectile vomiting

o Vomiting often occurs in the early morning before eating

Meningitis - fever

Hepatitis - jaundice

Reye’s syndrome - aspirin ingestion

Intussusception - intermittent or colicky abdominal pain

o palpable mass

Migraine - headache

o cyclic vomiting

Abdominal epilepsy - headache

o cyclic vomiting

Ketotic hypoglycemia

Drug intolerance/ toxicity

 

 

 10.      Compare and contrast the history and physical exam finding of a child presenting with functional constipation verses Hirschsprung’s disease.

Anonymous

Functional constipation: (No definite organic etiology), does not interfere w/ normal growth and development. Moderate to tense abdominal distension rarely occurs.

-Colicky abdominal pain may occur, especially while staining for stools, but abdominal tenderness is absent.

Page 37: Acute Gastroenteritis

 

Hirschsprung’s Disease: (congenital aganglionic megacolon), is one of the most common causes of neonatal abdominal obstruction and is the organic cause most commonly considered in eval. of constipation at any age.

-Full term newborns who fail to pass stool in the first 48 h of life, or who pass a meconium plug, should be observed closely. If stooling problems persist, an investigation w/ rectal biopsy is indicated.

-Vomiting and abdominal distension are sign and symptoms

-Failure to thrive, particularly when associated w/ constipation, may occur in pts w/ Hirschsprung’s disease.

Anonymous  Dershewitz pp. 597-598

Functional constipation does not interfere with normal growth and development, and an organic cause is absent.  Colicky abdominal pain occurs when straining to have a stool, but abdominal tenderness is absent.  Constipation can be caused by excessive milk intake, anticholinergics, lead toxicity, cathartic abuse, narcotics, and aluminum-based antacids.  60% of functional cases start in the 1st year, the rest start later.  The BM is usually at infrequent intervals; there could be withholding the effort to defecate, and also a positive soiling history.  The rectum can by dilated with stool at the ampulla.  A barium enema results in a normal evacuation time.  In the developmental stage of the infant a fissure may develop, while a toddle may resist having a BM.

Hirshsprung's Disease (congenital aganglionic megacolon): One of the most common causes of neonatal abdominal obstruction and is the organic cause most commonly considered in the evaluation of constipation at any age.  One-third of all cases remain undiagnosed until after 3 months of age, and 15-25% of cases are not diagnosed until after 5 years old.  Full-term newborns who fail to pass stool within the first 48 hours of life, or who pass a meconium plug, should be observed closely.  If stooling problems persist a rectal biopsy is indicated.  In infants or older children who Hirshsprung's dz is suspected a rectal manometry (tests function of anus and abnormality) or barium enema may be suggested for screening.  These kids are sometimes seen with failure to thrive that is associated with constipation.  Also moderate to tense abdominal distention can be present.  The child usually needs to have enemas, and has no soiling history.  The rectum is characteristically empty.  In barium enema the evacuation is prolonged over 24 hours.

Anonymous Derschwitz; 597-598

Hirschsprung’s Disease (congenital aganglionic megacolon) is one of the most common causes of neonatal abdominal obstruction and is the organic cause most commonly considered in the evaluation of constipation at any age.

Functional Constipation Hirschsprung’s Disease

Page 38: Acute Gastroenteritis

does not interfere with normal growth and development

abdominal distension is rare

colicky abdominal pain may occur, especially when straining for stools, but abdominal tenderness is absent

no vomiting usually

failure to thrive when associated with constipation is suggestive of Hirschsprung’s

moderate to tense abdominal distension often present

vomiting

failure to pass stool in the first 48 hours of life should be observed closely

Michelle Current Pediatric 16th Edition Page 639

  Retentive Constipation

Hirschsprung Disease

Onset 2-3 years At birthAbd. Distention Rare PresentNutrition & Growth Normal PoorSoiling & retentive Behavior

Intermittent or constant

Rare

Rectal Examination Ampulla full Ampulla maybe emptyRectal Biopsy Ganglion cells

presentGanglion cells absent

Rectal Manometry Normal rectoanal reflex

Nonrelaxation of internal anal sphincter after rectal distention

Barium Enema Distended rectum Narrow distal segment with proximal megacolon

Sung K, Current Pediatric p.639

  Retentive Constipation

Hirschsprung Disease

Onset 2-3 years At birthAbdominal distention Rare PresentNutrition and growth Normal PoorSoiling and retentive behavior

Intermittent or constant

Rare

Rectal exam Ampulla full Ampulla may be emptyRectal biopsy Ganglion cells

presentGanglion cells absent

Rectal manometry Normal rectoanal reflex

Nonrelaxation of internal anal sphincter after rectal distension

Barium enema Distended rectum Narrow distal segment with proximal

Page 39: Acute Gastroenteritis

megacolon

Anonymous

Functional constipation:

(No definite organic etiology), does not interfere w/ normal growth and development. Moderate to tense abdominal distension rarely occurs.

-Colicky abdominal pain may occur, especially while staining for stools, but abdominal tenderness is absent.

 

Hirschsprung’s Disease:

(Congenital aganglionic megacolon), is one of the most common causes of neonatal abdominal obstruction and is the organic cause most commonly considered in eval of constipation at any age.

-Full term newborns who fail to pass stool in the first 48 h of life, or who pass a meconium plug, should be observed closely. If stooling problems persist, an investigation w/ rectal biopsy is indicated.

-Vomiting and abdominal distension are sign and symptoms

-Failure to thrive, particularly when associated w/ constipation, may occur in pts w/ Hirschsprung’s disease.

 

11.      Identify the characteristic types of rectal bleeding, causes of rectal bleeding in children at different ages (including most common cause of rectal bleeding in children), and differential diagnosis.

Janelisa, CPDT 642

  INFANT CHILD 2-12yo ADOLESCENT >12yoPainless melena Duodenal ulcer,

duodenal duplication, ileal duplication, Meckel diverticulum, gastric heterotopia

Duodenal ulcer,

duodenal duplication, ileal duplication, Meckel diverticulum, gastric heterotopia

Duodenal ulcer, leiomyoma (sarcoma)

Melena with pain,

Necrotizing  enterocolitis,

Duodenal ulcer, hemobilia,

Duodenal ulcer, hemobilia, Crohn’s dz

Page 40: Acute Gastroenteritis

obstruction, peritonitis, perforation

intussusception, volvulus

intussusception, volvulus, ileal ulcer (isolated)

Hematochezia with diarrhea, crampy abdominal pain

Infectious colitis, pseudomembranous colitis, eosinophilic colitis, Hirschsprung enterocolitis

Infectious colitis, pseudomembranous colitis, granulomatous (Crohn) colitis, hemolytic-uremic syndrome, Henoch-Schonlein purpura, lymphonodular hyperplasia

Infectious colitis, pseudomembranous colitis, granulomatous (Crohn) colitis, hemolytic-uremic syndrome, Henoch-Schonlein purpura

Hematochezia without diarrhea or abdominal pain

Anal fissure, eosinophilic colitis, rectal gastric mucosa heterotopia, colonic hemangiomas

Anal fissure, solitary rectal ulcer, juvenile polyp, lymphonodular hyperplasia

Anal fissure, hemorrhoid, solitary rectal ulcer, colonic arteriovenous malformation

 

There is a similar table in Tintinalli 847. Blood represents local irritation or erosion in the majority of children, probably due to an anal fissure or could be related to food substances that have a red or melanotic coloration. Tintinalli 851 Examples of these are medications in red syrup, Kool-Aid, tomatoes, cranberry juice, beets, etc. Full list is in CPDT 640.

Anonymous  Derschewitz-pg. 614-615

Hematochezia- bright red, fresh blood from rectum. This is usually from rectum or colon

Melena- tarry black stool from upper tract bleed.

Currant Jelly- red blood mixed with mucous, usually distal small bowel or proximal colon-seen with intussusception or acute colitis.

Maroon stool- red and dark blood seen w bleeding from distal small bowel or proximal colon.

Flecks of red in formed stool- flecks along surface indicate bleeding fissure or anorectal lesion.

 

DIFFERENTIAL DIAGNOSIS

Newborn (birth to 1 mo.)-  milk protein allergy, maternal blood, volvulus, Hirshprung’s, coagulopathy, anal fissure, upper intestinal hemorrhage.

Page 41: Acute Gastroenteritis

Infant (1mo-1yr)-  milk allergy, anal fissure, intussusception, upper G-I hemorrhage, infectious enterocolitis, Meckles divertriculum, vascular malformation.

Childhood (1-12yrs)- anal fissure, infectious enterocolitis, juvenile polyp, intussusception, Meckles divertriculum, inflammatory bowel disease, Henoch-Schonlein pupura, hemolytic-uremic syndrome, upper G-I hemorrhage, vascular malformation.

Adolescent (>12yrs)- anal fissure, infectious enterocolitis, inflammatory bowel disease, Meckel’s divertriculum, Henoch-Schonlein pupura, hemolytic-uremic syndrome, vascular malformation, hemorroids, upper G-I hemorrhage.      

Anonymous Derschewitz; 614-615

Characteristic types of rectal bleeding

Hematochezia is the passage of fresh blood per rectum. Melena or black tarry stool reflect blood that has had a delayed transit time thru the GI

tract.

Currant Jelly Stool refers to a mixture of blood and mucus seen in ischemic or inflammatory lesions such as intestinal intussusception or acute colitis.

Maroon stool describes a mixture of red and dark blood seen with bleeding from the distal small bowel or proximal colon.

Formed stool with streaks or flecks of blood along its surface generally indicates a bleeding fissure or other anorectal lesion.

 

Differential Diagnosis/ most common cause - *The causes of RB must be considered with respect to the patient’s age and are listed in the approximate order of frequency in Table 137-1 (listed below).

Newborn

(Birth- 1 month)

Infant

(1 month – 1 year)

Childhood

(1-12 years)

Adolescent

(>12 yrs) milk protein

allergy

anal fissure

swallowed maternal blood

upper GI bleed

necrotizing

milk protein allergy

anal fissure

intussusception

upper GI bleed

infectious enterocolitis

Anal fissure

Infectious enterocolitis

Juvenile polyp

Intussusception

Meckel’s diverticulum

Anal fissure

Infectious enterocolitis

Idiopathic inflammatory bowel disease

Meckel’s

Page 42: Acute Gastroenteritis

enterocolitis

Hirschsprung’s enterocolitis

Midgut volvulus

coagulopathy

Meckel’s diverticulum

Vascular malformation

Idiopathic inflammatory bowel disease

Henoch-Schonlein purpura

Hemolytic-uremic syndrome

Upper GI bleed

Vascular malformation

diverticulum

Henoch-Schonlein purpura

Hemolytic-uremic syndrome

Vascular malformation

Hemorrhoids

Upper GI bleed

Brent K- Current Ped

Excellent table on pg 642 table 20-11

 

12.       Describe the important features of the history and physical of a child less than 3 months old with a presenting problem of vomiting that would help you distinguish between normal regurgitation and gastroesophageal reflux.

Janelisa, CPDT 614 Effortless postprandial spitting is the most common sx of gastroesophageal reflux. Ruminative behavior is sometimes a sx, as are dysphagia, colic after feedings, and neck contortions. Apnea can occur when a young infant changes position after feeding. Reflux can be associated with aspiration pneumonia, chronic cough, wheezing and asthma-like attacks. If severe enough, reflux may cause failure to thrive, esophagitis with hematemesis, occult blood loss, anemia, esophageal stricture, and inflammatory esophageal polyps. This is usually diagnosed in thriving infants <6mo from an upper GI series or prolonged monitoring of esophageal pH. I couldn’t find any info that specifically addressed the 3 mo age group.

Anonymous Dershwitz 566-567

Normal regurgitation - effortless expulsion of gastric contents, usually occurs in infants

Feeding problems - *(unsure what they are looking for here - see notes below)

GE reflux - return of gastric contents into esophagus. Severity influenced by pressure of lower esophageal sphincter and motility of the body of the esophagus.

Info. gained via Hx:  

Page 43: Acute Gastroenteritis

Diet - specific food problem?

            Medications?

            Weight loss - ? Organic lesion, other chronic problem?

            Quality and quantity of vomit

            Weight gain - over feeding?

            Presence of bile - obstructed distal to duodenum

            Undigested food  - ? Problem with esophagus (achalasia, stricture)

            Mucus - feeding problems in infants or pylorospasm

            Blood usually = mucosal ulceration of esophagus, stomach or duodenum

*Relationship to meals may help define cause - early a.m. vomiting (pre-meal, usually projectile) observed frequently in Pts. with intracranial hypertension or metabolic dz. Vomiting after meal usually (psych or PUD in older peds). Assoc. symptoms can be: fever, diarrhea, jaundice, dysphagia, and head trauma

Anonymous Derschewitz; 566-567

•        Normal regurgitation

o       Effortless expulsion of gastric contents that usually occurs in infancy due to immaturity of LES

•        Feeding problems

o       Mucus in the vomitus is suggestive of feeding problems

o       Improperly prepared formula can cause vomiting

o       Swallowed blood from excoriation on mom’s nipple can cause hematemesis

•        Gastroesophageal reflux

o       Undigested food in the vomitus may be present with disorders affecting the esophagus

o       Blood in vomitus can be related to ulceration of the esophagus

•        (nothing specific listed for GERD in kids)

Page 44: Acute Gastroenteritis

Brent K- Current Ped 614, Tintinalli 760

             Vomiting results from forceful contraction of the diaphragm and abdominal muscles, whereas regurgitation is independent of any effort and typically postprandial.  Typically GERD is harmless but in severe cases many cause failure to thrive esophagitis with hematemesis, occult blood loss, anemia, esophageal stricture and inflammatory polyps.  Aspiration, pneumonia, chronic cough, wheezing, and asthma-like attacks are responsible.  

            Dysphagia, colic after feedings and neck contortions (Sandifer syndrome) may occur.  

  

13.       Discuss the possible significance of the following in children:

                        Bilious vomiting

                        Bloody vomiting

                        Projectile vomiting

                        Abdominal pain associated with vomiting

                        Diarrhea associated with vomiting

                        ”Current jelly” stool

Janelisa, CPDT 614, 616

o          Bilious vomiting –  small intestinal obstruction

o          Bloody vomiting –  swallowed maternal blood in newborns, oropharyngeal lesions, nosebleed, peptic disease, bleeding disorders, foreign bodies, esophageal varices, Mallory-Weiss tear from prolonged vomiting

o          Projectile vomiting – pyloric stenosis; vomitus is rarely bilious but may be blood-streaked

o          Abdominal pain associated with vomiting – pain localized to RLQ suggests appendicitis; midline or diffuse pain suggests pancreatitis or generalized peritonitis

o          Diarrhea associated with vomiting – viral and bacterial gastroenteritis

o          ”Current jelly” stool – 626, 642 intussusception; most frequent cause of intestinal obstruction in first 2 years of life

Anonymous Dershewitz p 567

Page 45: Acute Gastroenteritis

Bilious vomiting- If bilious vomiting is present, the obstruction is below the ampulla of Vater.  Bilious emesis in a newborn must be evaluated immediately, and one should assume a bowel obstruction, until the appropriate diagnostic studies are performed.

Bloody vomiting- Swallowed blood from an excoriation on a mother’s breast may result in hematemesis, because blood irritates the child’s GI tract

Projectile vomiting- This may be due to immaturity of the lower esophageal sphincter.

Abdominal pain associated with vomiting- This may be due to appendicitis, volvulus, or Intussusception.

Diarrhea associated with vomiting- This is often associated with gastroenteritis.

”Currant jelly” stool- This is a clinical finding of Intussusception.

Anonymous Dershewitz pg 566-5 67

Bilious vomiting-bile in vomit, the presence of bile suggests an obstruction distal to the duodenum.  Bilious emesis in a newborn must be evaluated immediately, and one should assume a bowel obstruction until the appropriate diagnostic studies are performed.

Bloody vomiting-blood into vomitus or hematemesis usually indicates mucosal ulceration of the esophagus, stomach, or duodenum.  In neonates, swallowed blood from an excoriation on a mother's breast may result in hematemesis, because blood irritates the child's gastrointestinal tract.

Projectile vomiting-early morning vomiting before eating is observed frequently in patients with intracranial hypertension (in which vomiting is usually projectile) or in metabolic diseases.

Abdominal pain associated with vomiting-may be due to appendicitis, Volvulus, or intussception.  If the pain is intermittent and colicky or if an abdominal mass is palpable, the health-care provider should consider an intussusception and performing diagnostic as well as potentially therapeutic barium enema.

Diarrhea associated with vomiting-diary is often associated with vomiting in gastroenteritis.  

FYI-Whenever fever is present, otitis media, pneumonia, and urinary track infection should be considered.  John does suggest hepatitis or biliary tract obstruction, such as that seen with cholelithiasis or choledocholithiasis.

’Current jelly’ stool-I couldn't find this in any of our books, but I called my friendly neighborhood pediatrician and he said current jelly stool is caused by intussusception.

Brent K- Current Ped 614-15, 626, Tintinalli 760        

Bilious vomiting- intestinal obstruction

Page 46: Acute Gastroenteritis

Bloody vomiting- swallowed maternal blood, oropharyngeal lesions, nose bleed, peptic disease, bleeding disorders, foreign bodies, esophageal varices, and a possible Mallory-Weiss Tear after prolonged vomiting.   

Pojectile vomiting- pyloric stenosis or increased intracranial pressure.  

Abdominal pain with vomiting- obstruction, pancreatitis, peritonitis, gastritis, peptic disease, gallstones and hep.

Diarrhea and vomiting- viral or bacterial gastroenteritis.

Currant jelly stool- stool with blood and mucus.  Commonly resulting from intusseseption of the intestine causing obstruction.

Anonymous

Bilious vomiting- If bilious vomiting is present, the obstruction is below the ampulla of Vater.  Bilious emesis in a newborn must be evaluated immediately, and one should assume a bowel obstruction, until the appropriate diagnostic studies are performed.

Bloody vomiting- Swallowed blood from an excoriation on a mother’s breast may result in hematemesis, because blood irritates the child’s GI tract

Projectile vomiting- This may be due to immaturity of the lower esophageal sphincter.

Abdominal pain associated with vomiting- This may be due to appendicitis, volvulus, or Intussusception.

Diarrhea associated with vomiting- This is often associated with gastroenteritis.

”Currant jelly” stool- This is a clinical finding of Intussusception.

 

14.       What are the signs and symptoms of hernias and at what age do they most

            frequently occur?

Janelisa, CPDT 616, 627-8 Hiatal hernias are classified as paraesophageal, (rare in childhood; presents with pain, esophageal obstruction, or respiratory compromise) and sliding (more common in children; may have no sx or present with gastroesophageal reflux). Umbilical hernias are more common in premature and/or African-American infants. Small bowel may incarcerate; if fascial defect is <1cm in diameter, most spontaneously regress. Inguinal hernias are usually indirect, occur more frequently in boys (9:1) than girls, and present at birth or any time thereafter. Most cases are asymptomatic and present with a painless swelling. The swelling may be associated with coughing, long periods of standing, vomiting, abdominal distension, crying.

Page 47: Acute Gastroenteritis

Tintinalli 937 Incidence of inguinal hernias is 10-20 per 1000 live births, is much higher in premature and low-birth-weight infants, and is sometimes associated with fussiness, vomiting or other feeding problems.

Anonymous Dershewitz pg 639

The most common finding in groin hernias and hydroceles is a bulge or lump in the child's groin or scrotum that is noticed by parents.  The mass may be transient or it may be present constantly.  If the mass is or was present in the inguinal region or in the inguinal region and scrotum, is likely to be a hernia.  If a transient or persistent swelling occurred in the scrotum alone, it may be hydrocele.  The key fact is that an inguinal hernia causes a lump or mass above the inguinal ligament.  The examination for suspected hernia begins with the inspection of the groin to determine whether asymmetry exists in whether an obvious mass is present.  The examiner should ascertain that the testes are in their normal position, because in a descended or retractile testis can appear as a groin mass.  The child with lymphadenopathy or an inguinal abscess may show evidence of a recent infection, such as tenderness, fever, or leukocytosis.  The chief finding in a child with a hernia is a sausage-shaped mass originating at the internal inguinal ring and extending a variable distance down the inguinal canal.  A femoral hernia presents as a mass more medial than a typical hernia and may be seen high in the proximal thigh.

AGES-hernias are 6 to 15 times more common in boys than in girls, but in children younger than one year this gender disparity is less pronounced.  Greater than one-third of all hernias are diagnosed before six months of age, and one-half are diagnosed before the child's first birthday.  Ten percent to 15 percent of children, especially those younger than one year, have clinically apparent bilateral hernias.  If only one hernia is present, it is twice as likely to be on the right side as on the left.

Brent K- Current Ped 616, 618, 627-28         

 Hiatal Hernia- GERD, pain, esophageal obstruction, respiratory compromise.  

Umbilical- swelling around umbilicus.  More common in premature than full- term infants and more common in black than white infants.  

Inguinal- painless inguinal swelling.  Mass may retract when infant is cold, frightened or agitated from PE.  Most inguinal hernias are indirect occurring more often in boys than girls (9:1).  May be present at birth or appear later, at any age.  

Diaphragmatic hernia- respiratory distress usually present, scaphoid abdomen, decreased breath sounds with a displaced PMI, pulmonary HTN, mediastinal shift with vascular kinking.  Congenital defect occurring in newborns.

 

Page 48: Acute Gastroenteritis

15.       Describe the clinical features of a patient with an intestinal obstruction and the procedure usually used for diagnosis.

Janelisa, Tintinalli 848 Presents in infants and young children with pain (manifested by irritability), vomiting, and abdominal distension. Later progression will have absent or diminished bowel sounds. Flat and upright films of the abdomen will show dilated loops of bowel with air-fluid levels. A barium-enema with Hirschsprung catheter will help differentiate between Hirschsprung disease, malrotation, and colonic stenosis and also separate lower large bowel obstruction from upper small bowel obstruction.

Anonymous  Intestinal obstruction presents in infants and young children in the classic manner, with symptoms of pain (manifested by irritability); vomiting; dry mucous membranes; abdominal distension; and later, absence or diminished bowel movements. Diagnosis  Flat and upright films of the abdomen show dilated loops of bowel with air fluid levels. Tintinalli

Anonymous Nobel pg 977-978

Acute mechanical intestinal obstruction-obstruction of the bowel lumen, most commonly caused by (1) adhesions or hernias (small bowel), or (2) cancer or diverticulitis (Colon).

Clinical features-abdominal pain, cramping, is seen with intestinal obstruction.  Small bowel-periumbilical or upper abdominal-cramping-vomiting of bile and mucous (high obstruction) or fecal material (low obstruction).  Obstipation develops. Colon-lower abdominal or generalized-cramping-obstipation early. Vomiting late if at all.  Prior symptoms of underlying cause. Bates page 86-87

On auscultation and percussion-and increasing pitch or frequency of bowel sounds, or a marked diminution, indicates intestinal obstruction and and ileus, respectively. Bates page 682

Intermittent emesis occurs, obstipation, patient is restless and has episodes of agony, hyperactive bowel sounds are clinical features.

Procedure for DX-CXR upright check for free air under the diaphragm. Plain films are useful in dx Obstruction. Barium studies are helpful too in dx.

Brent K- Current Ped 619-22

 Depending on the location of the obstruction, meconium may or may not be passed.  Vomiting with bile, abdominal distention, reluctance to feed.  Procedures for Dx include abdominal x-ray and barium enema.